美文网首页
stolz公式的证明

stolz公式的证明

作者: 洛玖言 | 来源:发表于2019-10-02 15:02 被阅读0次

    stolz公式的证明

    O'Stolz定理是处理数列不定式极限的有力工具,一般用于*/∞型的极限(即分母趋于正无穷大的分式极限,分子趋不趋于无穷大无所谓)、0/0型极限(此时要求分子分母都以0为极限)。O'Stolz定理用于数列,它有函数形式的推广,这两个都可以认为是洛必达法则的离散版本。

    已知:

    • y_{n+1}>y_n\qquad(n=1,2,\cdots)
    • \displaystyle\lim_{n\to\infty}y_n=+\infty
    • \displaystyle\lim_{n\to\infty}\dfrac{x_{n+1}-x_n}{y_{n+1}-y_n}存在

    证:
    假定\displaystyle\lim_{n\to\infty}\dfrac{x_{n+1}-x_n}{y_{n+1}-y_n}=a\;.并且注意到 \;y_n\to\infty\;,可知对于任给的 \;\varepsilon>0\;,存在正整数N,使当 \;n>N\;时,恒有:
    \left|\dfrac{x_{n+1}-x_n}{y_{n+1}-y_n}-a\right|<\dfrac{\varepsilon}{2}\quad (且\;y_n>0).
    于是,当 \;n>N\;时,有:
    \dfrac{x_{N+2}-x_{N+1}}{y_{N+2}-y_{N+1}},\dfrac{x_{N+3}-x_{N+2}}{y_{N+3}-y_{N+2}},\cdots,\dfrac{x_{n}-x_{n-1}}{y_{n}-y_{n-1}},\dfrac{x_{n+1}-x_n}{y_{n+1}-y_n}
    都包含在\left(a-\frac{\varepsilon}{2},a+\frac{\varepsilon}{2}\right) 之内,又 y_{n+1}>y_n ,所以,这些分母都是正数,于是有:
    \left(a-\frac{\varepsilon}{2}\right)(y_{N+2}-y_{N+1})<x_{N+2}-x_{N+1}<\left(a+\frac{\varepsilon}{2}\right)(y_{N+2}-y_{N+1}),\\ \;\\ \left(a-\frac{\varepsilon}{2}\right)(y_{N+3}-y_{N+2})<x_{N+3}-x_{N+2}<\left(a+\frac{\varepsilon}{2}\right)(y_{N+3}-y_{N+2}),\\ \vdots\\ \left(a-\frac{\varepsilon}{2}\right)(y_{n+1}-y_{n})<x_{n+1}-x_{n}<\left(a+\frac{\varepsilon}{2}\right)(y_{n+1}-y_{n}),
    将这一些列的不等式相加,得:
    \left(a-\frac{\varepsilon}{2}\right)(y_{n+1}-y_{N+1})<x_{n+1}-x_{N+1}<\left(a+\frac{\varepsilon}{2}\right)(y_{n+1}-y_{N+1})
    即:
    \left(a-\frac{\varepsilon}{2}\right)<\dfrac{x_{n+1}-x_{N+1}}{y_{n+1}-y_{N+1}}<\left(a+\frac{\varepsilon}{2}\right)
    所以当 n>N\;时,恒有 \left|\dfrac{x_{n+1}-x_{N+1}}{y_{n+1}-y_{N+1}}-a\right|<\dfrac{\varepsilon}{2}\;,另外我们有,当 n>N\;时:
    \dfrac{x_n}{y_n}-a=\dfrac{x_{N+1}-ay_{N+1}}{y_n}+\left(1-\dfrac{y_{N+1}}{y_n}\right)\left(\dfrac{x_{n+1}-x_{N+1}}{y_{n+1}-y_{N+1}}-a\right)
    故有:\left|\dfrac{x_n}{y_n}-a\right|\leqslant\left|\dfrac{x_{N+1}-ay_{N+1}}{y_n}\right|+\dfrac{\varepsilon}{2}
    现取 N'>N\;,使当 n>N'\;,恒有:
    \dfrac{\left|x_{N+1}-ay_{N+1}\right|}{y_n}<\dfrac{\varepsilon}{2}
    于是当 n>N'\;时,恒有
    \left|\dfrac{x_n}{y_n}-a\right|<\varepsilon.
    由此可知,\displaystyle\lim_{n\to\infty}\dfrac{x_n}{y_n}=\lim_{n\to\infty}\dfrac{x_{n+1}-x_n}{y_{n+1}-y_n}=a


    应用

    就记录一些刚好有人问到的题目,或者突然碰到的题目


    Q1

    \lim_{n\to\infty}\dfrac{\displaystyle\sum_{k=1}^n\frac{1}{k}}{\ln n}

    Sol-1:
    \displaystyle\lim_{n\to\infty}\dfrac{\displaystyle\sum_{k=1}^n\frac{1}{k}}{\ln n}=\lim_{n\to\infty}\dfrac{1}{(n+1)[\ln(n+1)-\ln n]}\sim\lim_{n\to\infty}\dfrac{n}{n+1}=1

    Sol-2:
    \displaystyle\sum_{k=1}^n\frac{1}{k}=\ln n+\gamma+\varepsilon_n\,,\quad \varepsilon_n\to0
    \displaystyle\lim_{n\to\infty}\dfrac{\displaystyle\sum_{k=1}^n\frac{1}{k}}{\ln n}=\lim_{n\to\infty}\dfrac{\ln n+\gamma+\varepsilon_n}{\ln n}=1


    Q2

    \lim_{n\to\infty}\dfrac{1!+2!+\cdots+n!}{n!}

    Sol-1:
    \displaystyle\lim_{n\to\infty}\dfrac{1!+2!+\cdots+n!}{n!}=\lim_{n\to\infty}\dfrac{(n+1)!}{n\cdot n!}=\lim_{n\to\infty}\dfrac{n+1}{n}=1

    Sol-2:
    I=\displaystyle\dfrac{1!+2!+\cdots+n!}{n!}

    \displaystyle \dfrac{n!}{n!}\leqslant I\leqslant \dfrac{(n-2)(n-2)!+(n-1)!+n!}{n!}\\
    两边令 n\to\infty
    1\leqslant I\leqslant 1
    \displaystyle\therefore \lim_{n\to\infty}I=1


    Q3

    证明:
    \displaystyle\lim_{n\to\infty}a_n=a(有限数或正负无穷大).试证:
    \lim_{n\to\infty}\dfrac{a_1+a_2+\cdots+a_n}{n}=a

    Sol-1:
    \begin{aligned} \lim_{n\to\infty}\dfrac{a_1+a_2+\cdots+a_n}{n}=\lim_{n\to\infty}\dfrac{a_n}{n-(n-1)}=\lim_{n\to\infty}a_n=a \end{aligned}

    相关文章

      网友评论

          本文标题:stolz公式的证明

          本文链接:https://www.haomeiwen.com/subject/ksxwyctx.html